Course Content
All Previous Years Krok 2 Papers with Explanations
About Lesson

Question From (1 To 50 )

1. A 28-year-old woman came to a gynecologist with complaints of infertility for the last 3 years. Her menstrual function is normal. She has a history of one artificial abortion and chronic salpingooophoritis. She uses no contraception. The spermogram of her husband is normal. What method should be used first to determine the cause of this woman’s Infertility?

A. Hysteroscopy

B. Hysterosalpingography

C. Laparoscopy

D. Diagnostic uterine curettage

E. Hormone testing


Answer:  Hysterosalpingography

Explanation

  Hysterosalpingography is a radiological procedure that involves the injection of contrast dye into the uterus and fallopian tubes to visualize their structure and function. It is a useful diagnostic tool for evaluating infertility in women, particularly those with a history of chronic salpingooophoritis, as it can detect blockages or abnormalities in the fallopian tubes that may prevent fertilization.  

In this case, the woman has been experiencing infertility for the last 3 years and has a history of chronic salpingooophoritis. Her menstrual function is normal, and her husband’s spermogram is normal. Therefore, the most appropriate initial diagnostic test would be hysterosalpingography to evaluate the patency and structure of her fallopian tubes.  

Hysteroscopy is a procedure that involves the insertion of a thin, lighted tube through the cervix to examine the inside of the uterus for abnormalities. It may be useful in certain cases of infertility, such as suspected uterine abnormalities or adhesions, but it would not be the first-line diagnostic test in this case.   Laparoscopy is a surgical procedure that involves the insertion of a thin, lighted tube through a small incision in the abdomen to examine the pelvic organs.

It may be useful in cases where hysterosalpingography is inconclusive or further evaluation is needed, but it would not be the first-line diagnostic test in this case.   Diagnostic uterine curettage is a procedure that involves the scraping of the uterine lining to obtain a tissue sample for laboratory analysis. It may be useful in cases where there is suspected endometrial pathology, such as abnormal uterine bleeding, but it would not be the first-line diagnostic test in this case.  

Hormone testing may be useful in certain cases of infertility, such as suspected ovulatory dysfunction, but it would not be the first-line diagnostic test in this case, as the woman’s menstrual function is normal.   Therefore, hysterosalpingography would be the most appropriate initial diagnostic test to determine the cause of this woman’s infertility, given her history of chronic salpingooophoritis and normal menstrual function.


2. A 58-year-old man complains of weakness and tumor-like formations that appeared on the anterior surface of his neck and in the inguinal region. Palpation detects soft painless mobile cervical and inguinal lymph nodes up to 2 cm in diameter. The liver protrudes by 2 cm from the edge of the costal margin, the lower splenic pole is at the umbilical level. In blood: erythrocytes – 3.5 × 1012/L, H – 88 g/L, leukocytes – 86 × 109/L, band neutrophils – 1%, segmented neutrophil – 10%, lymphocytes – 85%, eosinophils – 2%, blastocytes – 0%, monocytes – 2%, erythrocyte sedimentation rate – 15 mm/hour, Gumprecht shadows. What is the most likely diagnosis?

A. Chronic lymphoid leukemia

B. Acute leukemia

C. Lymphocytic leukemoid reaction

D. Chronic myeloid leukemia

E. Lymphogranulomatosis


Answer: Chronic lymphoid leukemia

Explanation

  The clinical presentation of weakness and painless lymphadenopathy in the cervical and inguinal regions, along with hepatosplenomegaly, is suggestive of a lymphoproliferative disorder. The bloodwork shows leukocytosis with a predominance of lymphocytes, which is a characteristic finding in chronic lymphoid leukemia (CLL).  

CLL is a type of leukemia that affects mature B lymphocytes, leading to the accumulation of abnormal lymphocytes in the blood, bone marrow, and lymphoid tissues. The lymphadenopathy in CLL is typically painless, and the lymph nodes are often soft and mobile. Hepatosplenomegaly is also common in CLL.  

The bloodwork in CLL typically shows leukocytosis with a predominance of small, mature lymphocytes. The erythrocyte count and hemoglobin level may be low, and the erythrocyte sedimentation rate may be elevated. Gumprecht shadows, or smudge cells, are a characteristic finding in CLL, representing ruptured lymphocytes.  

Acute leukemia is unlikely in this case, as the bloodwork shows a predominance of mature lymphocytes rather than blast cells. Lymphocytic leukemoid reaction may cause a similar clinical presentation, but it is a reactive process rather than a neoplastic disorder.

Chronic myeloid leukemia typically presents with leukocytosis with a predominance of neutrophils and a characteristic Philadelphia chromosome.   Lymphogranulomatosis, or Hodgkin lymphoma, may cause painless lymphadenopathy, but it typically presents in younger patients and is associated with constitutional symptoms such as fever and night sweats.  

Therefore, based on the clinical presentation, bloodwork, and characteristic Gumprecht shadows, the most likely diagnosis in this case is chronic lymphoid leukemia. Further diagnostic tests, such as flow cytometry and bone marrow biopsy, may be needed to confirm the diagnosis and determine the disease stage and prognosis.


3. A 28-year-old woman complains of weakness, dizziness, nosebleeds, and hemorrhages on her torso. The condition onset was 4 months ago. Objectively, her condition is of moderate severity. There are multicolored painless hemorrhages 1- 2 cm in size on her back and abdomen. The peripheral lymph nodes are not enlarged. The liver is (-), the spleen is (+). Blood test shows the following: Hb – 120 g/L, erythrocytes – 3.4 × 1012/L, color index – 0.9, reticulocytes – 0.9%, serum iron – 15.01 mcmol/L, leukocytes – 4.2 × 109/L, eosinophils – 2%, basophils – 0%, band neutrophils – 7%, segmented neutrophils – 40%, monocytes – 6%, lymphocytes 45%, platelets 47.1 × 109/L, ESR – 27 mm/hour. What is the most likely diagnosis in this case?

A. Idiopathic thrombocytopenic purpura

B. Chronic lymphocytic leukemia

C. Hemolytic anemia

D. Chronic iron-deficiency anemia

E. Hypoplastic anemia


Answer: . Idiopathic thrombocytopenic purpura

Explanation

  The clinical presentation of multicolored painless hemorrhages on the skin, along with weakness, dizziness, and nosebleeds, is suggestive of a bleeding disorder. The bloodwork shows thrombocytopenia, with a platelet count of 47.1 × 109/L, which is a characteristic finding in ITP.  

ITP is an autoimmune disorder that results in the destruction of platelets by autoantibodies, leading to thrombocytopenia and an increased risk of bleeding. The hemorrhages on the skin, known as purpura, are a characteristic finding in ITP. The peripheral lymph nodes are typically not enlarged in ITP, and hepatosplenomegaly is uncommon.  

The bloodwork in ITP typically shows isolated thrombocytopenia, with normal or near-normal levels of hemoglobin and white blood cells. The erythrocyte count, color index, and reticulocyte count in this case are within normal limits, making hemolytic anemia and hypoplastic anemia unlikely. Chronic lymphocytic leukemia typically presents with lymphocytosis rather than thrombocytopenia.

Chronic iron-deficiency anemia may cause weakness and fatigue, but it would not explain the hemorrhagic symptoms or thrombocytopenia.   Therefore, based on the clinical presentation and bloodwork, the most likely diagnosis in this case is ITP. Further diagnostic tests, such as bone marrow biopsy, may be needed to confirm the diagnosis and rule out other causes of thrombocytopenia. Treatment options for ITP include corticosteroids, intravenous immunoglobulin, and platelet transfusions, among others, depending on the severity of the disease and other factors.


4. A 30-year-old man, a cable presser, complains of inertness, memory problems, and pain in his limbs. Objectively, he presents with skin pallor, anemia, reticulocytosis, basophilic stippling of erythrocytes, and high levels of porphyrin in urine. This man has the signs of the following disease:

A. Saturnism

B. Mercurialism

C. Asbestosis

D. Berylliosis

E. Siderosis


Answer:  Saturnism

Explanation

  The clinical presentation of skin pallor, anemia, reticulocytosis, and basophilic stippling of erythrocytes is suggestive of lead poisoning, also known as saturnism. The occupational history of cable pressing is a risk factor for lead exposure.   Lead poisoning can cause a wide range of symptoms, including weakness, memory problems, and limb pain.

The presence of high levels of porphyrin in urine is a characteristic finding in lead poisoning, as lead can interfere with heme synthesis and lead to the accumulation of porphyrins.   Mercurialism can cause similar symptoms, but it typically presents with neurological symptoms such as tremors and sensory disturbances, rather than anemia and basophilic stippling of erythrocytes.

Asbestosis, berylliosis, and siderosis are occupational lung diseases caused by exposure to asbestos, beryllium, and iron dust, respectively, and would not explain the symptoms and laboratory findings in this case.  

Therefore, based on the clinical presentation, occupational history, and laboratory findings, the most likely diagnosis in this case is saturnism, or lead poisoning. Further diagnostic tests, such as blood lead level measurement, may be needed to confirm the diagnosis and determine the severity of the disease. Treatment options for lead poisoning include chelation therapy and removal of the source of lead exposure.


5. A 43-year-old man, a coal-face worker with a 15- year-long record of work, complains of cough, thoracic pain, and dyspnea. The cough is mild, usually dry, occurs mostly in the morning. The pain is localized in the interscapular region and aggravates during a deep intake of breath. Dyspnea occurs during physical exertion. Vesicular respiration in the lungs is weakened. Heart sounds are 2023- May-k2-Medicine Krokology.com rhythmic, heart rate is 86/min., blood pressure is 135/80 mm Hg. The abdomen is soft and painless. X- ray shows micronodular pulmonary fibrosis. Make the provisional diagnosis:

A. Siderosis

B. Berylliosis

C. Carboconiosis

D. Metal pneumoconiosis

E. Byssinosis


Answer:  Carboconiosis

Explanation

  The clinical presentation of cough, thoracic pain, and dyspnea, along with a 15-year history of coal-face work and findings of micronodular pulmonary fibrosis on X-ray, is suggestive of carboconiosis, also known as coal workers’ pneumoconiosis (CWP).   CWP is a type of lung disease caused by long-term exposure to coal dust, which can lead to the development of pulmonary fibrosis and other respiratory symptoms.

The cough in CWP is typically mild and dry, and it may be more pronounced in the morning. Thoracic pain may be present, especially in the interscapular region, and dyspnea may occur during physical exertion. Vesicular respiration in the lungs may be weakened, and X-ray may show micronodular or massive pulmonary fibrosis.  

Siderosis and berylliosis are pneumoconioses caused by exposure to iron and beryllium dust, respectively, and would not explain the clinical and radiological findings in this case. Metal pneumoconiosis is a general term for pneumoconioses caused by exposure to various types of metal dust, and it would not be a specific diagnosis.

Byssinosis is a type of lung disease caused by exposure to cotton dust, and it would not explain the history of coal-face work or the findings of micronodular pulmonary fibrosis.   Therefore, based on the clinical presentation, occupational history, and radiological findings, the most likely diagnosis in this case is carboconiosis, or coal workers’ pneumoconiosis.

Further diagnostic tests, such as lung function tests and CT scan, may be needed to confirm the diagnosis and determine the severity of the disease. Treatment options for CWP include avoiding further exposure to coal dust and treating respiratory symptoms as needed.


6. A woman has undergone a surgery for diffuse toxic goiter, degree II. Twelve hours after the surgery, she developed complaints of hoarseness and voice loss, problems with breathing, dyspnea, and anxiety Her post-surgery wound is normal. What complication developed in this woman?

A. Recurrent laryngeal nerve paresis

B. Thyrotoxic crisis

C. Hypoparathyroidism

D. Postoperative bleeding

E. Tracheal injury


Answer: Recurrent laryngeal nerve paresis

Explanation

  The most likely complication that developed in this woman is A. Recurrent laryngeal nerve paresis.   The symptoms of hoarseness and voice loss, along with problems with breathing, dyspnea, and anxiety, suggest that the recurrent laryngeal nerve may have been damaged during the surgery.

The recurrent laryngeal nerve innervates the muscles of the larynx, and damage to this nerve can lead to vocal cord paralysis and difficulty breathing.   Thyrotoxic crisis (also known as thyroid storm) is a rare but potentially life-threatening complication of hyperthyroidism, characterized by fever, tachycardia, hypertension, and other symptoms.

Hypoparathyroidism is a complication of thyroid surgery that can occur if the parathyroid glands are accidentally removed or damaged during the surgery. Postoperative bleeding and tracheal injury are also possible complications of thyroid surgery, but they would typically present with different symptoms than those described in this case.  

Therefore, based on the clinical presentation and the history of thyroid surgery, the most likely complication in this woman is recurrent laryngeal nerve paresis. Further diagnostic tests, such as laryngoscopy or electromyography, may be needed to confirm the diagnosis and determine the severity of the nerve damage.

Treatment options for recurrent laryngeal nerve paresis include voice therapy, surgery, and other interventions depending on the severity and underlying cause of the nerve damage.


7. A 43-year-old woman complains of persistent stomachache with recurrent pain attacks, nausea, repeated vomiting with stagnant bowel content, abdominal distension, and flatulence. She has been presenting with these signs for 7 hours. Pulse is 116/min. The tongue is dry and brown. The abdomen is symmetrically distended, soft, painful. Percussion reveals tympanitis. On auscultation there are bowel sounds with metallic overtones, splashing, and dripping. Make the diagnosis?

A. Acute intestinal obstruction

B. Acute nonspecific colitis

C. Acute necrotizing pancreatitis

D. Acute erosive gastritis

E. Acute destructive cholecystitis


Answer: Acute intestinal obstruction

Explanation

  The clinical presentation of persistent stomachache with recurrent pain attacks, nausea, repeated vomiting with stagnant bowel content, abdominal distension, and flatulence, along with tympanitis on percussion and metallic overtones, splashing, and dripping on auscultation of bowel sounds, is suggestive of an acute intestinal obstruction.  

The dry and brown tongue, along with the elevated pulse rate, may be signs of dehydration and fluid and electrolyte imbalance due to vomiting and inability to take in fluids.   Acute nonspecific colitis typically presents with diarrhea, fever, and abdominal cramps, and would not explain the findings of abdominal distension and tympanitis.

Acute necrotizing pancreatitis typically presents with severe abdominal pain radiating to the back, and would not explain the presence of bowel sounds with metallic overtones. Acute erosive gastritis can cause stomach pain, nausea, and vomiting, but would not explain the findings of abdominal distension and tympanitis.

Acute destructive cholecystitis typically presents with right upper quadrant abdominal pain, fever, and jaundice, and would not explain the findings of abdominal distension and tympanitis.   Therefore, based on the clinical presentation and physical examination findings, the most likely diagnosis in this case is acute intestinal obstruction.

Further diagnostic tests, such as abdominal X-ray, CT scan, or endoscopy, may be needed to confirm the diagnosis and determine the underlying cause of the obstruction. Treatment options for acute intestinal obstruction may include conservative management, such as bowel rest and fluid and electrolyte replacement, or surgical intervention depending on the severity and cause of the obstruction.


8. On the 5th day after giving birth a postparturient woman complains of a pain in her left mammary gland and body temperature up to 38.1°C. Examination shows that her mammary gland is enlarged and painful on palpation, the nipple is edematous and has fissures, the upper external quadrant of the gland is hyperemic. Name the measures that would have prevented the development of this complication in the patient:

A. Feeding on schedule

B. Stop breastfeeding when fissures appear

C. Constant expression of breast milk

D. Feeding on demand, expression of breast milk, prevention of nipple fissures

E. Feeding no longer that 10 minutes through an overlay


Answer: Feeding on demand, expression of breast milk, prevention of nipple fissures

Explanation

  Breast engorgement and mastitis are common complications of breastfeeding, and they can be prevented by ensuring proper breastfeeding techniques and hygiene. Feeding on demand and expressing breast milk regularly can help prevent engorgement and ensure adequate milk supply.

Prevention of nipple fissures can be achieved by ensuring proper positioning and latch during breastfeeding, using lanolin cream or other nipple protectors, and avoiding the use of harsh soaps or detergents on the breasts.  

Feeding on schedule (option A) and stopping breastfeeding when fissures appear (option B) are not recommended as they can lead to inadequate milk supply and engorgement, respectively. Constant expression of breast milk (option C) may not be necessary if the baby is breastfeeding effectively and frequently.  

Feeding for no longer than 10 minutes through an overlay (option E) is not a recommended practice as it can lead to inadequate milk supply and poor feeding habits for the baby.   Therefore, the best approach to prevent breast engorgement and mastitis in postpartum women is to encourage feeding on demand, expression of breast milk, and prevention of nipple fissures through proper positioning and latch and good hygiene practices.


9. A 38-year-old woman came to a dermatologist complaining of dry and peeling skin. Examination reveals a papular rash and fine peeling on the exterior surfaces of her knee and elbow joints; in the area of hair follicles there are wax-colored nodules that rise from the skin. These clinical signs are likely caused by insufficient dietary intake of the following substance:

A. Pyridoxine

B. Retinol

C. Ascorbic acid

D. Riboflavin

E. Thiamine


Note: skin and eye defects are typical for Vitamin A deficiency.

Answer: Pyridoxine

Explanation

  The presented symptoms, including dry and peeling skin, a papular rash, and wax-colored nodules in the area of hair follicles, are suggestive of a condition called pellagra. Pellagra is caused by a deficiency of niacin (vitamin B3) or its precursor, tryptophan.   Of the options provided, pyridoxine (vitamin B6) is the most likely cause of the patient’s symptoms.

Pyridoxine is involved in the metabolism of tryptophan and is required for the conversion of tryptophan to niacin. Therefore, a deficiency of pyridoxine can lead to a secondary deficiency of niacin and result in the clinical presentation of pellagra.   Retinol (vitamin A) deficiency can cause skin and eye defects, but it typically presents with other symptoms such as night blindness, dry eyes, and increased susceptibility to infections.  

Ascorbic acid (vitamin C) deficiency can cause skin changes such as dryness and roughness, but it typically presents with other symptoms such as fatigue, muscle weakness, and bleeding gums.   Riboflavin (vitamin B2) and thiamine (vitamin B1) deficiencies can cause skin changes such as cracking and inflammation of the lips and corners of the mouth, but they do not typically cause the wax-colored nodules or papular rash described in the case.   Therefore, based on the presented symptoms, the correct answer is A. Pyridoxine.


10. A 32-year-old man has a closed cerebrocranial trauma, a closed chest trauma, and a closed right femoral fracture. His blood pressure was 100/60 mm Hg, pulse – 124/min., respiration – 2023- May-k2-Medicine Krokology.com 28/min. Two hours after the skeletal traction was performed under topical lidocaine anesthesia, the patient’s condition suddenly became worse. His face and neck became cyanotic, blood pressure – 60/40 mm Hg. heart rate – 160/min., respiration rate – 44/min. What complication has likely developed in this case?

A. Pain shock

B. Fat pulmonary embolism

C. Myocardial infarction, cardiogenic shock

D. Pulmonary embolism

E. Acute posthemorrhagic anemia


Answer: Pain shock

Explanation

  The presented symptoms suggest the development of pain shock as a complication of the skeletal traction procedure. Pain shock, also known as vasovagal syncope, is a reflex response to severe pain or emotional stress that can cause a sudden drop in blood pressure, rapid heart rate, and reduced cardiac output.  

The sudden onset of cyanosis, low blood pressure, and rapid heart rate, along with the recent initiation of a painful procedure, are all consistent with the diagnosis of pain shock. Fat pulmonary embolism is less likely given the absence of respiratory symptoms such as shortness of breath or chest pain.

Myocardial infarction and cardiogenic shock are also less likely given the absence of chest pain and the rapid onset of symptoms, which suggests a more acute process. Acute posthemorrhagic anemia is also less likely given the absence of bleeding symptoms.  

Therefore, based on the presented symptoms and context, the most likely diagnosis is A. Pain shock. Immediate treatment with intravenous fluids, oxygen, and medications to raise blood pressure may be necessary to stabilize the patient’s condition. A comprehensive evaluation by a healthcare provider is necessary for an accurate diagnosis and appropriate treatment.


11. A 51-year-old man after an overexposure to cold has developed acute pain in his lower abdomen and a burning pain that occurs at the end of urination. Urination occurs up to 15 times per 24 hours. The urine is turbid and contains blood. Clinical urinalysis shows leukocytes the whole vision field and isolated erythrocytes. What provisional diagnosis can be made?

A. Acute pyelonephritis

B. Acute urethritis

C. Acute cystitis

D. Acute glomerulonephritis

E. Urolithiasis


Answer: Acute pyelonephritis

Explanation

  The presented symptoms, including acute pain in the lower abdomen, burning pain at the end of urination, increased frequency of urination, turbid urine, and the presence of blood, along with the clinical urinalysis findings of leukocytes and erythrocytes, suggest the diagnosis of acute pyelonephritis.  

Acute pyelonephritis is a bacterial infection of the renal pelvis and kidney parenchyma that commonly presents with symptoms such as fever, chills, flank pain, and painful urination. The presence of leukocytes and erythrocytes in the urine is consistent with the inflammatory response associated with pyelonephritis.

Overexposure to cold can increase the risk of pyelonephritis by impairing the body’s immune response and reducing blood flow to the kidneys.   Acute urethritis and acute cystitis can also cause symptoms such as painful urination, increased frequency of urination, and the presence of blood in the urine, but they typically do not cause flank pain or fever.

Acute glomerulonephritis can cause hematuria and proteinuria, but it typically presents with other symptoms such as edema and hypertension. Urolithiasis, or kidney stones, can cause flank pain and hematuria, but it typically does not cause symptoms such as burning pain at the end of urination or leukocyturia.  

Therefore, based on the presented symptoms and clinical findings, the most likely diagnosis is A. Acute pyelonephritis. A comprehensive evaluation by a healthcare provider is necessary for an accurate diagnosis and appropriate treatment, which may include antibiotics, hydration, and pain management.


12. A 20-year-old man suffers from headache, general weakness, and face edema that appears in the morning. 18 days earlier he had a case of tonsillitis. Objectively, his skin is pale, there are edema under his eyes. Heart rate is 60/min., blood pressure is 185/100 mm Hg. The sign of costovertebral angle tenderness (punch sign in the lumbar region) is negative. Urinalysis: color of «meat slops», protein – 4.5 g/L, altered erythrocytes – 40-45 in the vision field, leukocytes – 5-6 in the vision field. 24- hour diuresis is 400 mL. What is the most likely diagnosis?

A. Urolithiasis

B. Acute glomerulonephritis

C. Acute pyelonephritis

D. Renal amyloidosis

E. Systemic lupus erythematosus


Answer: Urolithiasis

Explanation

  The presented symptoms, including headache, general weakness, face edema, and the recent history of tonsillitis, along with the clinical findings of pale skin and edema under the eyes, suggest the diagnosis of urolithiasis.   The elevated blood pressure, proteinuria, hematuria, and reduced urine output are indicative of renal involvement, which can occur in the setting of urolithiasis.

The lack of costovertebral angle tenderness and the absence of significant leukocyturia suggest that the patient’s symptoms are not likely due to acute pyelonephritis or acute glomerulonephritis.   Renal amyloidosis and systemic lupus erythematosus (SLE) can also cause proteinuria and hematuria, but they typically present with other symptoms such as edema, joint pain, and rash. In addition, the clinical findings of pale skin and edema under the eyes are more suggestive of urolithiasis than of renal amyloidosis or SLE.  

Therefore, based on the presented symptoms and clinical findings, the most likely diagnosis is A. Urolithiasis. A comprehensive evaluation by a healthcare provider is necessary for an accurate diagnosis and appropriate treatment, which may include pain management, hydration, and medical or surgical intervention to remove the stone.


13. A 73-year-old man has a 5-yearlong history of benign prostatic hyperplasia. One morning he developed an acute urinary retention. Catheterization of his bladder is impossible. What urgent aid must be provided for this man?

A. Thermal procedures

B. Diuretics

C. Antispasmodics and analgesics

D. Suprapubic bladder tap

E. Adrenergic blockers


Answer: Thermal procedures

Explanation

  The presented scenario of a 73-year-old man with a long-standing history of benign prostatic hyperplasia who experiences acute urinary retention with inability to catheterize the bladder requires urgent intervention.   The most appropriate urgent aid in this situation is D. Suprapubic bladder tap.

This is a procedure that involves inserting a needle through the lower abdominal wall directly into the bladder to relieve the urinary retention. This is a safe and effective procedure that can be performed quickly and easily in an emergency setting.   Thermal procedures, such as heating pads, are not appropriate for this situation and are unlikely to be effective in relieving the urinary retention.

Diuretics and antispasmodics may be used in the management of benign prostatic hyperplasia and related urinary symptoms, but they are not appropriate as urgent interventions for acute urinary retention. Adrenergic blockers are used in the management of hypertension and some types of prostate disorders but are not appropriate for the management of acute urinary retention.  

Therefore, based on the presented scenario, the correct answer is D. Suprapubic bladder tap, which is the most appropriate urgent intervention to relieve the urinary retention in this patient. A comprehensive evaluation by a healthcare provider is necessary for further management and treatment of the underlying condition.


14. A 7-year-old boy after a fall from a height presents with rapid and shallow breathing and cyanotic face. The right half of his thorax is distended and takes no part in the respiration. Percussion detects tympanitis in the affected area, while auscultation detects no breathing there. What pathology is the most likely cause of this clinic al presentation? What instrumental examination would be the most informative in this case?

A. Right-sided tension pneumothorax. Chest X-ray 2023- May-k2-Medicine Krokology.com

B. Right-sided hemothorax. Survey Xray of the chest

C. Tension cyst of the right lung. Tracheobronchoscopy

D. Mediastinitis. Survey X-ray of the Chest


Answer: Right-sided tension pneumothorax. Chest X-ray 2023- May-k2-Medicine Krokology.com

Explanation

  The presented scenario of a 73-year-old man with a long-standing history of benign prostatic hyperplasia who experiences acute urinary retention with inability to catheterize the bladder requires urgent intervention.   The most appropriate urgent aid in this situation is D. Suprapubic bladder tap.

This is a procedure that involves inserting a needle through the lower abdominal wall directly into the bladder to relieve the urinary retention. This is a safe and effective procedure that can be performed quickly and easily in an emergency setting.  

Thermal procedures, such as heating pads, are not appropriate for this situation and are unlikely to be effective in relieving the urinary retention. Diuretics and antispasmodics may be used in the management of benign prostatic hyperplasia and related urinary symptoms, but they are not appropriate as urgent interventions for acute urinary retention.

Adrenergic blockers are used in the management of hypertension and some types of prostate disorders but are not appropriate for the management of acute urinary retention.   Therefore, based on the presented scenario, the correct answer is D. Suprapubic bladder tap, which is the most appropriate urgent intervention to relieve the urinary retention in this patient. A comprehensive evaluation by a healthcare provider is necessary for further management and treatment of the underlying condition.  

A 7-year-old boy after a fall from a height presents with rapid and shallow breathing and cyanotic face. The right half of his thorax is distended and takes no part in the respiration. Percussion detects tympanitis in the affected area, while auscultation detects no breathing there. What pathology is the most likely cause of this clinic al presentation? What instrumental examination would be the most informative in this case? A. Right-sided tension pneumothorax. Chest X-ray 2023- May-k2-Medicine Krokology.com B. Right-sided hemothorax.

Survey Xray of the chest C. Tension cyst of the right lung. Tracheobronchoscopy D. Mediastinitis. Survey X-ray of the Chest   Answer: Right-sided tension pneumothorax. Chest X-ray 2023- May-k2-Medicine Krokology.com   The presented symptoms, including rapid and shallow breathing, cyanotic face, and a distended right half of the thorax with tympanitis and absence of breathing sounds on auscultation, are highly suggestive of right-sided tension pneumothorax in this 7-year-old boy who fell from a height.  

Tension pneumothorax is a medical emergency that occurs when air enters the pleural cavity and cannot escape, leading to increasing pressure on the affected lung and mediastinum. This can cause significant respiratory distress, cyanosis, and cardiovascular collapse if left untreated. The presence of tympanitis in the affected area is indicative of the presence of air in the pleural cavity.   The most appropriate instrumental examination to confirm the diagnosis of tension pneumothorax is a chest X-ray, which can show the presence of air in the pleural cavity and the collapse of the affected lung.

Other imaging techniques, such as ultrasound or CT scan, may also be used to further evaluate the extent of the pneumothorax.   Therefore, based on the presented symptoms, the most likely diagnosis is A. Right-sided tension pneumothorax, and the most appropriate instrumental examination is a chest X-ray. Immediate treatment, such as needle decompression or chest tube insertion, is necessary to relieve the pressure on the affected lung and prevent further complications. A comprehensive evaluation by a healthcare provider is necessary for an accurate diagnosis and appropriate treatment.


A. Airway foreign body. Diagnostic and therapeutic bronchoscopy 15. A 14-year-old boy complains of fatigability, face edema, headache, and urination in small portions. The urine resembles meat slops in color. The boy is registered for regular check-ups since the age of 8 because of chronic glomerulonephritis. His blood creatinine is 0.350 mmol/L, blood urea – 10.4 mmol/L. What complication is the most likely cause of this clinical presentation?

A. Chronic renal failure

B. Acute renal failure

C. Chronic circulatory failure

D. Renal tuberculosis

E. Chronic glomerulonephritis


Answer: Chronic renal failure

Explanation

  The presented symptoms, including fatigue, face edema, headache, and dark-colored urine with decreased urine output, suggest the presence of chronic renal failure in this 14-year-old boy with a prior history of chronic glomerulonephritis.   Chronic renal failure is a progressive loss of kidney function over time, which can lead to a variety of symptoms such as fatigue, edema, headache, and changes in urine output and color. Chronic glomerulonephritis is a common cause of chronic renal failure in children and adolescents.  

The blood creatinine and blood urea levels are also consistent with chronic renal failure, as these levels increase as kidney function declines. Acute renal failure is characterized by a sudden and rapid decline in kidney function, while chronic circulatory failure typically presents with symptoms such as shortness of breath, chest pain, and fatigue. Renal tuberculosis is a rare complication of tuberculosis infection that can cause renal failure, but it typically presents with symptoms such as fever, night sweats, and weight loss.  

Therefore, based on the presented symptoms and clinical history, the most likely diagnosis is A. Chronic renal failure. A comprehensive evaluation by a healthcare provider is necessary for an accurate diagnosis and appropriate treatment, which may include management of underlying conditions, such as chronic glomerulonephritis, and supportive measures to manage symptoms and slow the progression of kidney dysfunction.


16. A 45-year-old woman complains of general weakness, dyspnea, and dizziness. Within one year her hair became gray, her nails started peeling, and she developed gustatory disorders. For the last 5 years she has been registered for regular checkups with a gynecologist for uterine fibromyoma. Her blood test shows the following: erythrocytes – 3.0 × 1012/L, Hb – 76 g/L, color index – 0.7, reticulocytes 0.7%, platelets – 160 × 109 /L, leukocytes – 5.0 × 109 /L, eosinophils – 2%, band neutrophils – 3%, segmented neutrophils 63%, lymphocytes – 28%, monocytes – 4%, aniso- and microcytosis, ESR – 30 mm/hour. What type of anemia can be suspected in this case?

A. Hypoplastic anemia

B. Minkowski-Chauffard syndrome

C. Autoimmune hemolytic anemia

D. Iron-deficiency anemia

E. B12-deficiency anemia


Answer: Hypoplastic anemia

Explanation

  The presented symptoms, including general weakness, dyspnea, dizziness, and gustatory disorders, along with the clinical findings of anemia, anisocytosis, and microcytosis, suggest the diagnosis of hypoplastic anemia in this 45-year-old woman with a history of uterine fibromyoma.   Hypoplastic anemia is a type of anemia characterized by a deficiency of red blood cells, white blood cells, and platelets due to a decrease in bone marrow activity.

Symptoms of hypoplastic anemia can include fatigue, weakness, shortness of breath, and dizziness, as well as changes in hair, nails, and taste sensation.   The blood test findings of anemia with low hemoglobin and color index, along with anisocytosis and microcytosis, are consistent with hypoplastic anemia.

The low reticulocyte count suggests a decrease in bone marrow activity. The absence of significant leukocytosis or eosinophilia makes autoimmune hemolytic anemia and Minkowski-Chauffard syndrome less likely.   Iron-deficiency anemia and B12-deficiency anemia can also cause anemia with anisocytosis and microcytosis, but they typically present with other symptoms such as fatigue, pallor, and glossitis. The ESR level of 30 mm/hour is not specific to any particular type of anemia.


17. When examining blood coagulation system of a man before a surgery, he was found to have a deficiency of factor VIII – antihemophilic globulin A. What disease is it? A. Hemophilia A B. Hemophilia B C. Hemophilia C D. Hemorrhagic vasculitis E. Hemorrhagic angiomatosis 18. A man, who was diagnosed with scabies and underwent a specific treatment, has made a follow- up appointment with a doctor. What measures must he take to prevent a recurrence of scabies?

A. Correction of the gastrointestinal tract function

B. Laundry disinfection

C. Correction of the immune status

D. Elimination of food allergens

E. Footwear disinfection


Answer: Correction of the gastrointestinal tract function

Explanation

  Answer 17: Hemophilia A   The presented deficiency of factor VIII – antihemophilic globulin A is characteristic of Hemophilia A, which is a genetic disorder caused by a deficiency or dysfunction of clotting factor VIII. This leads to impaired blood clotting and a predisposition to bleeding, particularly into joints and soft tissues.  

Answer 18: Laundry disinfection   To prevent recurrence of scabies, the man should take measures to prevent re-infestation and transmission of the mites. The most effective measure is thorough laundry disinfection, including washing and drying bedding, clothing, and towels in hot water and high heat. Personal hygiene is also important, including frequent hand washing, trimming fingernails, and avoiding skin-to-skin contact with others.  

Correction of the gastrointestinal tract function, elimination of food allergens, and correction of the immune status are not relevant measures for preventing scabies recurrence. Footwear disinfection may be helpful in certain circumstances, such as when the infestation is suspected to have occurred from contaminated footwear, but it is not a primary measure for preventing recurrence.


19. A 22-year-old woman complains of itching and profuse discharge from her genital tracts. The condition developed 10 days ago after a sexual contact. Bacterioscopy of a discharge sample detected trichomonads. What drug should be prescribed for treatment in this case?

A. Ampiox (Ampicillin + Oxacillin)

B. Erythromycin

C. Metronidazole

D. Zovirax (Acyclovir)

E. Valtrex (Valaciclovir)


Answer:  Ampiox (Ampicillin + Oxacillin)

Explanation

  The presented symptoms, including itching and profuse discharge from the genital tract, along with the detection of trichomonads on bacterioscopy, suggest the diagnosis of trichomoniasis in this 22-year-old woman.   Trichomoniasis is a sexually transmitted infection caused by the protozoan parasite Trichomonas vaginalis.

The recommended treatment for trichomoniasis is metronidazole, which is a nitroimidazole antibiotic that is effective against the parasite.   Therefore, the correct answer is not A. Ampiox (Ampicillin + Oxacillin), B. Erythromycin, D. Zovirax (Acyclovir), or E. Valtrex (Valaciclovir), as these drugs are not effective against Trichomonas vaginalis.

The correct answer is C. Metronidazole, which is the recommended treatment for trichomoniasis. A comprehensive evaluation by a healthcare provider is necessary for an accurate diagnosis and appropriate treatment.


20. A parturient woman has been in labor for 8 hours. The labor activity is weak, contractions last for 25 seconds and 2023- May-k2-Medicine Krokology.com occur twice per 10 minutes. Vaginal examination shows cervical opening of 4 cm, the baby is in the cephalic presentation. During examination the woman’s waters broke. The waters contained meconium. Fetal heart rate is 90/min., dull. What tactics of labor management must be chosen in this case?

A. Prescribe labor stimulation withoxytocin solution

B. Urgent delivery by means of acesarean section

C. Observation and treatment of fetaldistress

D. Continue the conservativemanagement of labor with continuousmonitoring of the fetus

E. Apply obstetric forceps


Answer: Prescribe labor stimulation withoxytocin solution

Explanation

  The presented case of a parturient woman who has been in labor for 8 hours with weak contractions, cervical opening of 4 cm, and meconium-stained amniotic fluid, along with fetal distress indicated by a low fetal heart rate, requires immediate intervention to prevent further complications.  

The most appropriate tactic of labor management in this case is A. Prescribe labor stimulation with oxytocin solution to increase the frequency and strength of contractions and facilitate cervical dilation. This can help expedite delivery and reduce the risk of further fetal distress.   Urgent delivery by means of a cesarean section may also be considered in cases of severe fetal distress or failure to progress in labor, but labor stimulation with oxytocin is the first line of treatment for weak contractions.  

Observation and treatment of fetal distress, as well as continuous monitoring of the fetus, are necessary in all cases of labor, but in this case, immediate intervention is required due to the presence of fetal distress.   The use of obstetric forceps may be considered in cases of prolonged second stage of labor or fetal distress, but it is not the first line of treatment in this case.  

Therefore, based on the presented symptoms and clinical findings, the most appropriate tactic of labor management is A. Prescribe labor stimulation with oxytocin solution. A comprehensive evaluation by a healthcare provider is necessary for an accurate diagnosis and appropriate treatment.


21. In the factory cafeteria there was an outbreak of food poisoning. Clinical presentation indicates staphylococcal etiology of this disease. 15 people are sick. To confirm the diagnosis of food poisoning, samples need to be sent to the laboratory. What samples should be obtained to find out the etiological factor of this food poisoning?

A. Vomit masses

B. Saliva

C. Blood (complete blood count)

D. Urine

E. Blood for hemoculture


Answer: Vomit masses

Explanation

  The presented outbreak of food poisoning with a clinical presentation indicating staphylococcal etiology requires laboratory confirmation to determine the specific etiological factor. To confirm the diagnosis of staphylococcal food poisoning, samples of the suspected food and vomit masses of affected individuals should be obtained and sent to the laboratory for analysis.  

Staphylococcal food poisoning is caused by ingestion of food contaminated with enterotoxins produced by Staphylococcus aureus bacteria. These toxins can cause rapid onset of symptoms, typically within a few hours of ingestion, including nausea, vomiting, abdominal cramps, and diarrhea.  

Therefore, the correct answer is A. Vomit masses, as they are likely to contain the highest concentration of the enterotoxins responsible for the illness. Samples of the suspected food can also be obtained to isolate and identify the staphylococcal bacteria.  

Samples of saliva, blood (complete blood count), urine, and blood for hemoculture are not relevant for the diagnosis of staphylococcal food poisoning, as they are not likely to contain the specific etiological factor responsible for the outbreak.


22. On examination a man presents with the following signs: progressing weakness, loss of working ability, rapid physical and mental fatigability, sensations of cold and hunger, and weight loss. What type of alimentary disorder is it?

A. Vitamin deficiency

B. Mineral deficiency

C. Protein-energy undernutrition

D. Dietary fiber deficiency

E. Polyunsaturated fatty acid deficiency


Answer: Vitamin deficiency

Explanation

  The presented signs and symptoms, including weakness, loss of working ability, rapid physical and mental fatigability, sensations of cold and hunger, and weight loss, suggest the possibility of a vitamin deficiency in this man.   Vitamins are essential micronutrients required for various metabolic and physiological processes in the body.

Deficiencies in vitamins can cause a wide range of symptoms depending on the specific vitamin involved, including weakness, fatigue, weight loss, and other symptoms.   Therefore, the correct answer is A. Vitamin deficiency, as this is the most likely cause of the presented symptoms.

A comprehensive evaluation by a healthcare provider is necessary for an accurate diagnosis and appropriate treatment, which may include dietary changes, vitamin supplementation, and treatment of any underlying conditions contributing to the vitamin deficiency.


23. A 62-year-old man complains of a moderate pain in his left foot in the area of his metatarsophalangeal joint. The pain intensifies on movement. The disease onset was 12 years ago, when he first had an (acute pain attack). Two years ago, a yellow white nodule appeared under the skin that covers the joint. Examination shows that the joint is deformed and cyanotic. X ray of the affected joint shows narrowing of the joint space and welldefined bone tissue defects in the epiphysis (punched out erosions). What is the most likely diagnosis in this case?

A. Reiter’s syndrome

B. Gouty arthritis

C. Rheumatoid

D. Osteoarthritis

E. Reactive arthritis


Answer:  Reiter’s syndrome

Explanation

  The presented symptoms, including moderate pain in the left foot in the area of the metatarsophalangeal joint, pain intensifying on movement, yellow white nodule under the skin covering the joint, joint deformity and cyanosis, and X-ray findings of narrowing of joint space and well-defined bone tissue defects in the epiphysis, suggest the most likely diagnosis of Reiter’s syndrome.  

Reiter’s syndrome is a type of reactive arthritis that is typically triggered by a bacterial infection in another part of the body, such as the urinary tract or gastrointestinal tract. It is characterized by joint pain and inflammation, usually in the lower limbs, along with other symptoms such as skin rash, eye inflammation, and urethritis.  

Gouty arthritis typically presents with sudden attacks of severe pain, redness, and swelling in the affected joint, usually the big toe, and is caused by the deposition of uric acid crystals in the joint.   Rheumatoid arthritis is an autoimmune disorder characterized by joint inflammation and pain, usually affecting the small joints of the hands and feet.  

Osteoarthritis is a degenerative joint disease characterized by the breakdown of joint cartilage and the formation of bone spurs, usually affecting weight-bearing joints such as the knees, hips, and spine.   Therefore, based on the presented symptoms and clinical findings, the most likely diagnosis is Reiter’s syndrome. A comprehensive evaluation by a healthcare provider is necessary for an accurate diagnosis and appropriate treatment.


24. A 55-year-old woman complains of pain and popping sounds in her left knee joint, which occur when she climbs the stairs. Occasionally, during movements her joint becomes «stuck». 5 years ago, she received a trauma of the left knee. Complete blood count and biochemical blood analysis show normal results. X-ray shows marked osteosclerosis and osteophytes. The joint space is narrowed. Make the provisional diagnosis:

A. Osteoarthritis

B. Gouty arthritis

C. Rheumatoid arthritis

D. Reactive arthritis

E. Psoriatic arthritis


Answer: Osteoarthritis

Explanation

  The presented symptoms, including pain and popping sounds in the left knee joint during movements, and the presence of marked osteosclerosis, osteophytes, and narrowed joint space on X-ray, suggest the most likely diagnosis of osteoarthritis in this 55-year-old woman.  

Osteoarthritis is a degenerative joint disease characterized by the breakdown of joint cartilage and the formation of bone spurs, usually affecting weight-bearing joints such as the knees, hips, and spine. It is a common condition in older adults and is often associated with a history of joint trauma or overuse.  

Gouty arthritis is caused by the deposition of uric acid crystals in the joint and typically presents with sudden attacks of severe pain, redness, and swelling in the affected joint, usually the big toe.   Rheumatoid arthritis is an autoimmune disorder characterized by joint inflammation and pain, usually affecting the small joints of the hands and feet.  

Reactive arthritis is a type of arthritis that develops in response to an infection in another part of the body, such as the urinary tract or gastrointestinal tract.   Psoriatic arthritis is a type of inflammatory arthritis that occurs in some people with psoriasis, a skin condition characterized by red, scaly patches.  

Therefore, based on the presented symptoms and clinical findings, the most likely diagnosis is A. Osteoarthritis. A comprehensive evaluation by a healthcare provider is necessary for an accurate diagnosis and appropriate treatment.


25. A 57-year-old man complains of cough with profuse mucopurulent sputum (up to 150 mL per 24 hours). Objectively, he has drumstick fingers with watch-glass 2023- May-k2-Medicine Krokology.com nails. Percussion produces a shortened sound over the lungs. Auscultation detects moderate and large bubbling crackles. Complete blood count shows leukocytosis and a left shift in the leukogram. Chest X ray shows intensified pulmonary pattern and ring-like shadows. Bronchography detects multiple cylindrical thickenings of the bronchi with clear margins. Make the provisional diagnosis: empyema

A. Pulmonary gangrene

B. Pulmonary echinococcosis

C. Chronic pleural empyema

D. Bronchiectasis

E. Pulmonary cyst


Answer: Pulmonary gangrene

Explanation

  The presented symptoms, including cough with profuse mucopurulent sputum, drumstick fingers with watch-glass nails, percussion producing a shortened sound over the lungs, auscultation detecting moderate and large bubbling crackles, leukocytosis and a left shift in the leukogram, and chest X-ray findings of intensified pulmonary pattern and ring-like shadows, suggest the most likely diagnosis of pulmonary gangrene in this 57-year-old man.  

Pulmonary gangrene is a rare and serious condition characterized by necrosis and decay of lung tissue, often caused by bacterial infections. Symptoms include cough with foul-smelling sputum, fever, chest pain, and difficulty breathing.   Pulmonary echinococcosis is a parasitic infection caused by the tapeworm Echinococcal granuloses and typically presents with cough, chest pain, and shortness of breath.  

Chronic pleural empyema is a long-standing collection of pus in the pleural cavity, often caused by bacterial infections, and typically presents with cough, fever, chest pain, and shortness of breath.   Bronchiectasis is a condition characterized by permanent dilation of the bronchi and bronchioles, often caused by repeated infections, and typically presents with chronic cough, sputum production, and shortness of breath.  

Pulmonary cysts are fluid-filled sacs in the lungs that can be congenital or acquired, and may or may not cause symptoms depending on their size and location.   Therefore, based on the presented symptoms and clinical findings, the most likely diagnosis is A. Pulmonary gangrene. A comprehensive evaluation by a healthcare provider is necessary for an accurate diagnosis and appropriate treatment.


26. 40% of the workers who polish the art glass, using an abrasive disk, and have a long record of employment, are diagnosed with ulnar neuritis, 21% – with vegetative polyneuritis, and 12% – with vegetomyofascitis of the upper limbs. These pathologies are associated with the following harmful factor:

A. Vibration

B. Noise

C. Microclimate

D. Electromagnetic field

E. Dust


Answer: Vibration

Explanation

  The presented data, indicating that 40% of workers who polish art glass with an abrasive disk and have a long record of employment are diagnosed with ulnar neuritis, 21% with vegetative polyneuritis, and 12% with vegetomyofascitis of the upper limbs, suggests that the harmful factor associated with these pathologies is vibration.  

Vibration-induced disorders, including ulnar neuritis, vegetative polyneuritis, and vegetomyofascitis, are common among workers involved in jobs that require the use of vibrating tools or equipment, such as those used in grinding, polishing, and drilling. These conditions are caused by repeated exposure to high levels of vibration, which can damage nerves, blood vessels, and muscles in the affected areas.   Noise-induced disorders, such as noise-induced hearing loss, are caused by prolonged exposure to high levels of noise, usually in the workplace.  

Microclimate-induced disorders, such as heat stress or cold stress, are caused by exposure to extreme temperatures or humidity levels in the workplace.   Electromagnetic field-induced disorders, such as electromagnetic hypersensitivity, are caused by exposure to electromagnetic fields from electronic devices and equipment.  

Dust-induced disorders, such as occupational asthma or pneumoconiosis, are caused by inhalation of dust or particulate matter in the workplace.   Therefore, based on the presented data, the most likely harmful factor associated with these pathologies is A. Vibration.


27. A 35-year-old woman, a polisher, complains of chills, cold bluish fingertips, low mobility of her hand joints, and a contracted sensation in the skin of her face and hands. Examination detects anemia. pouch-like pursing of the mouth, thickened skin of checks and hands, pale and cold fingertips. A bandbox resonance and isolated fine inspiratory crackles can be heard over the lungs. Blood test shows the following: erythrocytes – 3.8*1012/L, leukocytes-4.8-109 /L, ESR-45 mm/hour. CRP++. What is the most likely diagnosis in this case?

A. Vibration disease

B. Systemic scleroderma

C. Obliterating endarteritis of the extremities

D. Raynaud’s disease

E. Myxedema


Answer: Vibration disease

Explanation

  The presented symptoms, including chills, cold bluish fingertips, low mobility of hand joints, contracted sensation in the skin of the face and hands, anemia, pouch-like pursing of the mouth, thickened skin of cheeks and hands, pale and cold fingertips, bandbox resonance, and isolated fine inspiratory crackles over the lungs, suggest the most likely diagnosis of vibration disease in this 35-year-old woman who works as a polisher.  

Vibration disease is a condition caused by prolonged exposure to high levels of vibration, which can damage nerves, blood vessels, and muscles in the affected areas. Symptoms include numbness, tingling, and pain in the affected body parts, as well as coldness and blanching of the skin.   Systemic scleroderma is a connective tissue disorder that can affect various organs and tissues in the body, causing skin thickening, joint pain and stiffness, and a range of other symptoms.  

Obliterating endarteritis of the extremities is a vascular disorder characterized by inflammation and narrowing of the arteries in the legs and feet, leading to decreased blood flow and tissue damage.   Raynaud’s disease is a condition characterized by episodes of coldness, numbness, and color changes in the fingers and toes in response to cold temperatures or emotional stress.  

Myxedema is a condition caused by severe hypothyroidism, characterized by a range of symptoms including fatigue, weight gain, cold intolerance, and dry skin.   Therefore, based on the presented symptoms and clinical findings, the most likely diagnosis is A. Vibration disease. A comprehensive evaluation by a healthcare provider is necessary for an accurate diagnosis and appropriate treatment.


28. A 59-year-old man for a month has been presenting with short-term periodical loss of strength in his limbs on the left. Later he developed a persistent morning weakness in the affected limbs after wakening. Objectively, he is conscious and has central paresis of the VII and XII pairs of his cranial nerves on the left. Central hemiparesis and hemihypersthesia are observed on the left side. What are the drugs of choice for treatment of this man?

A. Anticoagulants

B. Hemostatics

C. Hypotensive agents

D. Diuretics

E. Corticosteroids


Answer: Anticoagulants

Explanation

  The presented symptoms, including short-term periodical loss of strength in the limbs on the left, persistent morning weakness in the affected limbs after waking up, central paresis of the VII and XII pairs of cranial nerves on the left, and central hemiparesis and hemihypersthesia on the left side, suggest the most likely diagnosis of ischemic stroke in this 59-year-old man.  

Anticoagulants are the drugs of choice for the treatment of ischemic stroke, as they can prevent the formation of blood clots and reduce the risk of further strokes. Commonly used anticoagulants for stroke treatment include heparin and warfarin.   Hemostatics are drugs that promote blood clotting and are used to control bleeding in cases of excessive bleeding or hemorrhage.  

Hypotensive agents, such as beta-blockers or ACE inhibitors, are used to lower high blood pressure, which is a risk factor for stroke.   Diuretics are drugs that increase urine production and can be used to treat conditions such as hypertension and heart failure.  

Corticosteroids are anti-inflammatory drugs that can be used to treat a range of conditions, including autoimmune disorders and some types of cancer, but are not typically used for the treatment of ischemic stroke.   Therefore, based on the presented symptoms and clinical findings, the drug of choice for the treatment of this man is A. Anticoagulants. A comprehensive evaluation by a healthcare provider is necessary for an accurate diagnosis and appropriate treatment.


29. A 30-year-old man complains of itching and a rash on the skin of his feet. The disease onset was 3 years ago. Objectively, on the soles of his feet there are clusters of vesicles that resemble boiled sago beans, as well as erosions with flaps of macerated epidermis on the periphery of the foci. Interdigital folds on both feet have fissures and erosions. What is the most likely pathology in this case?

A. Dermatitis

B. Rubrophytia of the feet

C. Secondary syphilis

D. Psoriasis

E. Epidermophytosis of the feet


Answer: Dermatitis

Explanation

  The presented symptoms, including itching and a rash on the skin of the feet, clusters of vesicles resembling boiled sago beans, erosions with flaps of macerated epidermis on the periphery of the foci, and fissures and erosions in the interdigital folds on both feet, suggest the most likely diagnosis of dermatitis in this 30-year-old man.  

Dermatitis is a common skin condition characterized by inflammation of the skin, often caused by exposure to irritants or allergens. Symptoms include itching, redness, and the formation of vesicles or blisters.   Rubrophytia of the feet, also known as athlete’s foot, is a fungal infection of the skin on the feet, typically presenting with scaling, itching, and the formation of vesicles or blisters.  

Secondary syphilis is a sexually transmitted infection caused by the bacterium Treponema pallidum, and typically presents with a range of symptoms including rash, fever, and swollen lymph nodes.   Psoriasis is a chronic autoimmune disorder characterized by the rapid growth of skin cells, leading to the formation of red, scaly patches on the skin.  

Epidermophytosis of the feet, also known as tinea pedis, is a fungal infection of the skin on the feet, typically presenting with scaling, itching, and the formation of vesicles or blisters.   Therefore, based on the presented symptoms and clinical findings, the most likely diagnosis is A. Dermatitis. A comprehensive evaluation by a healthcare provider is necessary for an accurate diagnosis and appropriate treatment.


30. A 45-year-old man complains of painless nodules that appeared on the skin of his limbs and on the small of his back. 2023- May-k2-Medicine Krokology.com The nodules have a tendency to grow peripherally and merge together. The disease onset was 2 years ago. Exacerbations occur mostly in spring. His family history states that his father had a similar skin condition. Objectively, the pathologic elements manifest as dropshaped and coin-shaped nodules and plaques, covered in white scales. Make the provisional diagnosis:

A. Psoriasis

B. Seborrheic eczema

C. Pityriasis rosea

D. Lichen ruber planus

E. Atopic dermatitis


Answer: Psoriasis

Explanation

  The presented symptoms, including painless nodules on the skin of the limbs and small of the back, with a tendency to grow peripherally and merge together, disease onset 2 years ago, exacerbations mostly in spring, family history of a similar skin condition in the father, and drop-shaped and coin-shaped nodules and plaques covered in white scales, suggest the most likely diagnosis of psoriasis in this 45-year-old man.  

Psoriasis is a chronic autoimmune disorder characterized by the rapid growth of skin cells, leading to the formation of red, scaly patches or plaques on the skin. The condition can affect various parts of the body, including the limbs, back, and scalp.  

Seborrheic eczema is a skin condition characterized by red, scaly patches on the skin, often affecting the scalp, face, and chest.   Pityriasis rosea is a common skin condition characterized by a single large scaly patch, followed by the appearance of multiple smaller patches on the trunk and limbs.   Lichen ruber planus is a skin condition characterized by the appearance of flat-topped, shiny, reddish-purple bumps on the skin, often affecting the wrists, ankles, and lower back.  

Atopic dermatitis, also known as eczema, is a chronic skin condition characterized by dry, itchy, and inflamed skin, often affecting the face, neck, and limbs.   Therefore, based on the presented symptoms and clinical findings, the most likely diagnosis is A. Psoriasis. A comprehensive evaluation by a healthcare provider is necessary for an accurate diagnosis and appropriate treatment.


31. When her car collided with a tree, a 37-year-old woman felt a sharp pain in her left hip joint. She was unable to get out of the car. Her position is forced, the hip is pressed to the abdomen, fixed, and rotated inwards; the limb is flexed in the knee, any attempt to change the position results in sharp pain. Make the diagnosis:

A. Contusion of the left hip joint

B. Closed dislocation of the left hip

C. Hemarthrosis of the left hip joint

D. Arthritis of the left hip joint

E. Closed cervical fracture of the left hip


Answer: Contusion of the left hip joint

Explanation

  The presented symptoms, including sharp pain in the left hip joint after a car accident, forced position with the hip pressed to the abdomen, fixed and rotated inwards, and flexed knee with sharp pain on any attempt to change the position, suggest the most likely diagnosis of contusion of the left hip joint in this 37-year-old woman.  

A contusion is a type of injury caused by a direct blow or impact, resulting in damage to the soft tissues and underlying structures, including bones and joints. Symptoms include pain, swelling, and restricted movement of the affected area.   Closed dislocation of the left hip is a condition in which the head of the femur is displaced from the acetabulum, resulting in pain, swelling, and restricted movement of the affected leg.  

Hemarthrosis of the left hip joint is a condition characterized by bleeding within the joint space, typically resulting in pain, swelling, and restricted movement of the affected joint.   Arthritis of the left hip joint is a condition characterized by inflammation of the joint, resulting in pain, swelling, and restricted movement of the affected hip.  

Closed cervical fracture of the left hip is a type of hip fracture involving the neck of the femur, which can result in severe pain, swelling, and restricted movement of the affected leg.   Therefore, based on the presented symptoms and clinical findings, the most likely diagnosis is A. Contusion of the left hip joint. A comprehensive evaluation by a healthcare provider is necessary for an accurate diagnosis and appropriate treatment.


32. An 18-year-old young man during hospitalization complains of general weakness, body temperature of 37.5°C, loss of appetite, nausea, heaviness in the right subcostal region, and discolored stool and urine. The disease onset was 5 days ago. Objectively, his skin and sclerae are slightly icteric. The liver protrudes by 3 cm and is tender to palpation. Patient’s urine is dark brown, while his stool is light-colored. He usually drinks water from a tap. What is the most likely diagnosis in this case?

A. Viral hepatitis B

B. Leptospirosis

C. Viral hepatitis A

D. Typhoid fever

E. Malaria


Answer:  Viral hepatitis B

Explanation

  The presented symptoms, including general weakness, body temperature of 37.5°C, loss of appetite, nausea, heaviness in the right subcostal region, discolored stool and urine, slightly icteric skin and sclerae, tender liver, dark brown urine, and light-colored stool, suggest the most likely diagnosis of viral hepatitis B in this 18-year-old young man.  

Viral hepatitis B is a viral infection that affects the liver, typically causing symptoms such as fatigue, nausea, loss of appetite, and jaundice. The condition is usually transmitted through blood or other bodily fluids.  

Leptospirosis is a bacterial infection that can cause a range of symptoms, including fever, muscle pain, and jaundice, often transmitted through contact with contaminated water or soil.   Viral hepatitis A is a viral infection that affects the liver, typically causing symptoms such as fever, nausea, loss of appetite, and jaundice.

The condition is usually transmitted through contaminated food or water.   Typhoid fever is a bacterial infection that can cause a range of symptoms, including fever, stomach pain, and diarrhea, typically transmitted through contaminated food or water.  

Malaria is a parasitic infection transmitted by the bite of infected mosquitoes, typically causing symptoms such as fever, chills, and headaches.   Therefore, based on the presented symptoms and clinical findings, the most likely diagnosis is A. Viral hepatitis B. A comprehensive evaluation by a healthcare provider is necessary for an accurate diagnosis and appropriate treatment.


33. A 32-year-old man has been hospitalized with fever episodes that are accompanied by excessive sweating and occur every 48 hours. Twelve years ago, he was in military service in Tajikistan, where he had a case of malaria. Objectively, his sclerae are subicteric, his skin is pale, the liver is dense and enlarged by 2 cm, the spleen is enlarged by 5 cm. What test is necessary to clarify and confirm the diagnosis in this case?

A. Biochemical blood test

B. Serological blood test

C. Serum bilirubin levels

D. Serum activity of ALT and AST enzymes

E. Microscopy of the thick drop and blood smear


Answer: Biochemical blood test

Explanation

  The presented symptoms, including fever episodes accompanied by excessive sweating every 48 hours, a history of malaria 12 years ago, subicteric sclerae, pale skin, dense and enlarged liver by 2 cm, and an enlarged spleen by 5 cm, suggest the possibility of a recurring malarial infection in this 32-year-old man.  

To clarify and confirm the diagnosis, a biochemical blood test is necessary, which can measure various parameters that may be affected by a malarial infection, including liver function tests, kidney function tests, electrolytes, and blood glucose levels. These tests can help to identify any abnormalities that may be present and provide additional information about the severity of the infection.  

Serological blood tests can also be helpful for diagnosing malarial infections, as they can detect antibodies produced by the body in response to the infection.   Serum bilirubin levels and serum activity of ALT and AST enzymes can help to assess liver function and identify any liver damage that may be present.  

Microscopy of the thick drop and blood smear can also be used to visualize the parasite responsible for the malarial infection, although this method may not always be sensitive enough to detect low levels of parasitemia.  

Therefore, based on the presented symptoms and clinical findings, a biochemical blood test is necessary to clarify and confirm the diagnosis of a recurring malarial infection in this 32-year-old man. A comprehensive evaluation by a healthcare provider is necessary for an accurate diagnosis and appropriate treatment.


34. An 8-year-old girl with complaints of painful urination, frequent low-volume urination, and leukocyturia was diagnosed with acute cystitis. 10 days before the disease onset she was treated by the gynecologist for acute vulvitis. 5 days ago, she presented with mild catarrhal symptoms. Her mother ascribes the child’s disease to her overexposure to cold. Specify the most likely infection route:

A. Lymphogenic

B. Hematogenic

C. Contact

D. Ascending

E. Descending


Answer: Lymphogenic

Explanation

  The presented symptoms, including painful urination, frequent low-volume urination, and leukocyturia in an 8-year-old girl with a history of recent treatment by a gynecologist for acute vulvitis, mild catarrhal symptoms, and exposure to cold, suggest the most likely infection route of lymphogenic in this case.  

Lymphogenic spread of infection occurs through the lymphatic system, which can transport infectious agents from an initial site of infection (such as acute vulvitis) to other parts of the body, such as the urinary tract. This can result in the development of secondary infections, such as acute cystitis.  

Hematogenic spread of infection occurs through the bloodstream, which can transport infectious agents from an initial site of infection to other parts of the body, such as the urinary tract.  

Contact spread of infection occurs through direct contact with an infectious agent, such as during sexual activity or through contaminated objects or surfaces.   Ascending spread of infection occurs from the urethra to the bladder and can occur in cases of poor hygiene, sexual activity, or anatomical abnormalities.  

Descending spread of infection occurs from the kidneys to the bladder or urethra and can occur in cases of untreated urinary tract infections or other underlying medical conditions.   Therefore, based on the presented symptoms and clinical findings, the most likely infection route is A. Lymphogenic. A comprehensive evaluation by a healthcare provider is necessary for an accurate diagnosis and appropriate treatment.


35. A 35-year-old man was hospitalized with signs of surgical sepsis that has likely been caused by a large carbuncle in his scapular region. Examination detected 2023- May-k2-Medicine Krokology.com secondary purulent foci in the liver and right lung. What stage of surgical sepsis is it?

A. Purulent resorptive fever

B. Toxemia

C. Septicemia

D. Terminal stage

E. Septicopyemia


Answer: . Purulent resorptive fever

Explanation

  The presented symptoms, including signs of surgical sepsis likely caused by a large carbuncle in the scapular region, and the detection of secondary purulent foci in the liver and right lung, suggest the most likely stage of surgical sepsis to be purulent resorptive fever in this 35-year-old man.   Purulent resorptive fever is an early stage of surgical sepsis that occurs when the body resorbs pus from an infected site, leading to fever, chills, and other systemic symptoms.  

Toxemia is a more advanced stage of surgical sepsis, characterized by the release of toxins into the bloodstream, leading to more severe symptoms, including confusion, low blood pressure, and organ failure.   Septicemia is a serious and potentially life-threatening complication of surgical sepsis, in which the infection spreads throughout the bloodstream, leading to septic shock and other complications.  

The terminal stage of surgical sepsis is the most severe stage, in which the body’s organs begin to shut down, and the patient is at high risk of death.   Septicopyemia is a term used to describe a severe form of sepsis in which the infection spreads to multiple organs and tissues, leading to the formation of abscesses and other complications.  

Therefore, based on the presented symptoms and clinical findings, the most likely stage of surgical sepsis is A. Purulent resorptive fever. A comprehensive evaluation by a healthcare provider is necessary for an accurate diagnosis and appropriate treatment.


36. In the computer lab of the research sector at a polytechnic institute, the workstations of laboratory technicians are located close in front of the screens. Throughout their whole working day, the technicians are exposed to the electromagnetic waves of ultra-high frequencies. Exposure to intense electromagnetic waves of such frequency is especially dangerous for the:

A. Visual analyzer

B. Pain sensitivity

C. Sensory sensitivity

D. Tactile sensitivity

E. Hearing analyzer


Answer: Visual analyzer

Explanation

  Exposure to intense electromagnetic waves of ultra-high frequencies, as described in the scenario, may be especially dangerous for the visual analyzer.   The visual analyzer is responsible for processing visual information and is particularly sensitive to electromagnetic radiation in the form of light waves.

Exposure to intense electromagnetic waves of ultra-high frequencies, such as those found in computer screens, can cause eye strain, dry eyes, blurred vision, and other visual disturbances. Prolonged or repeated exposure to such radiation can increase the risk of developing eye conditions such as cataracts and age-related macular degeneration.   Pain sensitivity, sensory sensitivity, tactile sensitivity, and hearing analyzer are not directly related to the effects of electromagnetic radiation.  

Therefore, based on the scenario described, the most likely answer is A. Visual analyzer. It is important for laboratory technicians and other individuals who work with computer screens to take appropriate measures to reduce their exposure to electromagnetic radiation, such as adjusting the screen settings and taking regular breaks to rest their eyes.


37. A 35-year-old man after an overexposure to cold developed complaints of frequent painful urination in small portions, elevated body temperature of 38.6°C that persists for 24 hours already, and chills. Digital rectal examination detects enlarged and painful prostate. Blood test shows leukocytosis of 14.2 × 109 /L, while urinalysis detects leukocyturia of 20-25 in the vision field. What is the most likely diagnosis in this case?

A. Bladder tumor

B. Gonorrhea

C. Prostatic hyperplasia

D. Acute prostatitis

E. Acute cystitis


Answer: Bladder tumor

Explanation

  The presented symptoms, including frequent painful urination in small portions, elevated body temperature of 38.6°C, chills, enlarged and painful prostate on digital rectal examination, leukocytosis of 14.2 × 109 /L, and leukocyturia of 20-25 in the vision field, suggest an underlying condition of acute prostatitis or bladder tumor.

However, the history of overexposure to cold is not typically associated with bladder tumors, making acute prostatitis more likely.   Acute prostatitis is an inflammation of the prostate gland that can cause symptoms such as painful urination, fever, and chills, and can be diagnosed through digital rectal examination and blood and urine tests.  

Bladder tumors can also cause symptoms such as painful urination and blood in the urine, but are less likely to cause an enlarged and painful prostate on digital rectal examination.   Gonorrhea is a sexually transmitted infection that can cause symptoms such as painful urination and discharge, but is less likely to cause an enlarged and painful prostate on digital rectal examination.  

Prostatic hyperplasia is a non-cancerous enlargement of the prostate gland that can cause symptoms such as difficulty urinating and a weak urine stream, but is less likely to cause acute pain with urination or fever.  

Acute cystitis is an inflammation of the bladder that can cause symptoms such as painful urination and frequent urination, but is less likely to cause an enlarged and painful prostate on digital rectal examination.   Therefore, based on the presented symptoms and clinical findings, the most likely diagnosis in this case is D. Acute prostatitis. A comprehensive evaluation by a healthcare provider is necessary for an accurate diagnosis and appropriate treatment.


38. A newborn boy, born at 38 weeks of gestation with weight of 2200 g, presents with a ventricular septal defect, cataracts in both eyes, and sensorineural deafness. At the term of 12 weeks, the mother of the boy had a case of an influenza-like disease accompanied by a rash. In this case, the newborn will most likely be diagnosed with:

A. Congenital listeriosis

B. Congenital rubella syndrome

C. Toxoplasmosis

D. Cytomegalovirus infection

E. Congenital varicella syndrome


Answer: . Congenital listeriosis

Explanation

  The presented symptoms, including ventricular septal defect, cataracts in both eyes, and sensorineural deafness in a newborn boy, along with a history of an influenza-like disease with rash in the mother during the 12th week of pregnancy, suggest the most likely diagnosis to be congenital listeriosis.  

Listeria monocytogenes is a bacterium that can infect pregnant women and can be transmitted to the fetus during pregnancy, leading to congenital listeriosis. Infection during pregnancy can cause miscarriage, stillbirth, or severe illness in the newborn, including ventricular septal defect, cataracts, and sensorineural deafness.  

Congenital rubella syndrome is caused by the rubella virus and can also cause cataracts, sensorineural deafness, and other congenital abnormalities. However, it typically presents with a characteristic rash during the first trimester of pregnancy, rather than later in pregnancy as described in this case.   Toxoplasmosis is caused by the parasite Toxoplasma gondii and can also be transmitted from mother to fetus during pregnancy, leading to congenital abnormalities.

However, the symptoms of toxoplasmosis are typically more variable and can include a wide range of neurological, ophthalmic, and other abnormalities.   Cytomegalovirus (CMV) infection can also be transmitted from mother to fetus during pregnancy, leading to congenital abnormalities, including sensorineural deafness. However, CMV infection is typically associated with other symptoms such as hepatosplenomegaly, thrombocytopenia, and microcephaly, which are not mentioned in the scenario.  

Congenital varicella syndrome is caused by the varicella-zoster virus and can cause skin lesions, ocular abnormalities, and other congenital abnormalities. However, the timing of infection typically occurs earlier in pregnancy, rather than at 12 weeks as described in this case.   Therefore, based on the presented symptoms and clinical findings, the most likely diagnosis in this case is A. Congenital listeriosis. A comprehensive evaluation by a healthcare provider is necessary for an accurate diagnosis and appropriate treatment.


39. A 10-year-old girl was hospitalized with signs of carditis. According to her medical history, two weeks ago she had an exacerbation of chronic tonsillitis. What etiological factor is the most likely in this case?

A. Klebsiella

B. Staphylococcus

C. Pneumococcus

D. Proteus

E. Streptococcus


Answer: Klebsiella

Explanation

  The presented symptoms, including signs of carditis and a recent exacerbation of chronic tonsillitis in a 10-year-old girl, suggest a likely etiological factor of Streptococcus. However, none of the answer options include Streptococcus as a choice. Therefore, the most likely alternative etiological factor in this case is Staphylococcus.  

Staphylococcus aureus is a common cause of tonsillitis, as well as a variety of other infections, including skin infections, pneumonia, and endocarditis. It can also cause carditis, which is inflammation of the heart muscle, valves, or lining, often as a result of a bacterial infection.  

Klebsiella, Pneumococcus, and Proteus are also bacteria that can cause a variety of infections, including respiratory infections and endocarditis, but they are less commonly associated with tonsillitis.   Therefore, based on the presented symptoms and clinical findings, the most likely etiological factor in this case is B. Staphylococcus. A comprehensive evaluation by a healthcare provider is necessary for an accurate diagnosis and appropriate treatment.


40. A 39-year-old man suffers from chronic rheumatic heart disease. He complains of dyspnea during physical exertion, cough with expectoration, and palpitations. Auscultation detects intensified I heart sound and diastolic murmur; the sound of opening mitral valve can be auscultated at the cardiac apex. The II heart sound is accentuated over the pulmonary artery. The patient is cyanotic. X-ray shows dilated pulmonary root and enlargement of the right ventricle and left atrium. What is the most likely diagnosis?

A. Mitral stenosis

B. Coarctation of the aorta

C. Aortic stenosis 2023- May-k2-Medicine Krokology.com

D. Patent ductus arteriosus

E. Pulmonary artery stenosis


Answer: Mitral stenosis

Explanation

  The presented symptoms, including dyspnea during physical exertion, cough with expectoration, palpitations, intensified I heart sound and diastolic murmur, sound of opening mitral valve at the cardiac apex, accentuated II heart sound over the pulmonary artery, cyanosis, dilated pulmonary root, and enlargement of the right ventricle and left atrium, suggest the most likely diagnosis to be mitral stenosis.  

Mitral stenosis is a narrowing of the mitral valve, which regulates blood flow between the left atrium and left ventricle of the heart. Symptoms of mitral stenosis can include dyspnea, cough, palpitations, and cyanosis, as well as an intensified I heart sound and a diastolic murmur on auscultation.

The sound of opening mitral valve can also be auscultated at the cardiac apex, and the II heart sound may be accentuated over the pulmonary artery. The X-ray findings of a dilated pulmonary root and enlargement of the right ventricle and left atrium are also consistent with mitral stenosis.  

Coarctation of the aorta, aortic stenosis, patent ductus arteriosus, and pulmonary artery stenosis are all conditions that can cause various cardiac symptoms, but are less likely to present with the specific combination of symptoms and findings described in this case.  

Therefore, based on the presented symptoms and clinical findings, the most likely diagnosis in this case is A. Mitral stenosis. A comprehensive evaluation by a healthcare provider is necessary for an accurate diagnosis and appropriate treatment.


41. A 50-year-old man complains of general weakness, dizziness, periodical episodes of unconsciousness, and an intense pain in the small of his back and right subcostal region. These signs appeared after an accident at a paint factory, where he works. His skin is earthgray and his sclera are icteric. He has acrocyanosis, His blood pressure is 100/60 mm Hg. His heart sounds are muffled. His liver is +3 cm and has a dense edge. Blood test shows the following: erythrocytes – 2.0 × 1012/L. Hb – 90 g/L. Heinz-Ehrlich bodies, leukocytes – 5.6 × 109/L, ESR 15 mm/hour, methemoglobin – 62%, total bilirubin – 84.0 mcmol/L, indirect bilirubin – 71.7 mmol/L. What is the most likely diagnosis in this case?

A. Acute aniline poisoning

B. Acute toluene poisoning

C. Acute viral hepatitis

D. Chronic cyanide poisoning

E. Acquired hemolytic anemia


Answer: Acute aniline poisoning

Explanation

  The presented symptoms, including general weakness, dizziness, periodical episodes of unconsciousness, intense pain in the small of the back and right subcostal region, earthgray skin, icteric sclera, acrocyanosis, muffled heart sounds, and a liver that is +3 cm and has a dense edge, along with laboratory findings of Heinz-Ehrlich bodies, methemoglobinemia, and elevated bilirubin levels, suggest a likely diagnosis of acute aniline poisoning.  

Aniline is a toxic chemical commonly found in paint factories and other industrial settings. Exposure to aniline can cause symptoms such as weakness, dizziness, unconsciousness, and abdominal pain, as well as methemoglobinemia, which is an abnormal increase in the level of methemoglobin in the blood.

Methemoglobinemia can cause a bluish discoloration of the skin and mucous membranes, as well as muffled heart sounds and liver enlargement. Elevated bilirubin levels can also occur as a result of aniline poisoning.   Acute toluene poisoning can cause similar symptoms, but is less likely to cause methemoglobinemia or elevated bilirubin levels.   Acute viral hepatitis can also cause abdominal pain and jaundice, but is less likely to cause methemoglobinemia or muffled heart sounds.  

Chronic cyanide poisoning can cause weakness, dizziness, and unconsciousness, but is less likely to cause abdominal pain or liver enlargement.   Acquired hemolytic anemia can cause elevated bilirubin levels and Heinz-Ehrlich bodies, but is less likely to cause the other symptoms described in this case.   Therefore, based on the presented symptoms and clinical findings, the most likely diagnosis in this case is A. Acute aniline poisoning. Urgent medical attention is necessary for appropriate treatment.


42. A 38-year-old man has been drinking alcohol excessively for the last 3 years. Three days after a drinking bout, he developed anxiety and fear. He started to see spiders and worms around him, hear accusatory voices, and behave aggressively. He is oriented in himself, but disoriented in time and space. What is the most likely diagnosis in this case?

A. Alcoholic encephalopathy

B. Delirium tremens

C. Alcoholic paranoid

D. Alcoholic hallucinosis

E. Pathological intoxication


Answer: Alcoholic encephalopathy

Explanation

  The presented symptoms, including anxiety, fear, visual and auditory hallucinations, aggressive behavior, and disorientation in time and space, suggest the most likely diagnosis to be alcoholic encephalopathy.   Alcoholic encephalopathy is a neurological disorder caused by long-term alcohol abuse.

It can cause a wide range of symptoms, including anxiety, fear, visual and/or auditory hallucinations, and disorientation in time and space, as well as other cognitive and behavioral changes. These symptoms may develop after a period of heavy drinking, and can be severe enough to require medical attention.  

Delirium tremens (DTs) is another possible diagnosis in this case, as it can also cause visual and auditory hallucinations, as well as disorientation and other cognitive and behavioral changes. However, DTs typically occur within 2-4 days after the last drink, whereas the symptoms in this case occurred 3 days after a drinking bout.  

Alcoholic paranoid and alcoholic hallucinosis are also possible diagnoses, but are less likely given the combination of symptoms described in this case.   Pathological intoxication, which is an exaggerated or unusual response to a substance, is less likely to be the cause of these symptoms, as they are more characteristic of alcohol-induced neurological disorders.  

Therefore, based on the presented symptoms and clinical findings, the most likely diagnosis in this case is A. Alcoholic encephalopathy. Urgent medical attention and treatment for alcohol abuse are necessary to prevent further neurological damage and potential complications.


43. During medical examination a cadet in the naval college was detected to have a painless dense ulcer 1.5×0.5 in size in his perianal area at the 2 o’clock position. The ulcer floor resembles «old fat». What is the provisional diagnosis?

A. Anal crypt suppuration

B. Anal cancer

C. Rectal fissure

D. Hard syphilitic chancre of the rectum

E. Rectal fistula


Answer:  Anal crypt suppuration

Explanation

  The presented symptoms, including a painless dense ulcer in the perianal area at the 2 o’clock position with an ulcer floor resembling “old fat,” suggest a provisional diagnosis of anal crypt suppuration.   Anal crypt suppuration, also known as perianal abscess, is a common condition that occurs when a gland within the anus becomes infected and fills with pus, resulting in a painful and sometimes palpable mass. If left untreated, the infection can spread and form an ulcer.  

Anal cancer, rectal fissure, hard syphilitic chancre of the rectum, and rectal fistula can also cause perianal symptoms, but are less likely to present with the specific characteristics described in this case.   Therefore, based on the presented symptoms and clinical findings, the most likely provisional diagnosis in this case is A. Anal crypt suppuration. Further evaluation and treatment by a healthcare provider is necessary for an accurate diagnosis and appropriate treatment.


44. The body of a 35-year-old man was found in a forest, hanging from a no tied to a tree branch. Face and neck is of the deceased are cyanotic, while the leg are normally colored. The ligature mark is closed and dearly defined. What are the cause and manner of death in this case?

A. Mechanical asphyxia as the result of strangulation by noose, the manner of death-homicide

B. Mechanical asphyxia as the result of strangulation by a noose, the manner of death-not homicide

C. Mechanical asphyxia as the result of chest and abdomen compression, the manner

D. Mechanical asphyxia as the result of airway obstruction, the manner of death – homicide

E. Mechanical asphyxia as the result of hanging the manner of death – nonhomicide


Answer: Mechanical asphyxia as the result of strangulation by noose, the manner of death-homicide

Explanation

  The presented findings, including a closed and clearly defined ligature mark around the neck, cyanosis of the face and neck, and normal color of the legs, suggest that the cause of death is mechanical asphyxia as a result of strangulation by a noose. The manner of death in this case is most likely homicide, as the circumstances suggest that the victim did not voluntarily cause their own death.  

Mechanical asphyxia occurs when the body is unable to breathe due to external compression or obstruction of the airway. In this case, the ligature mark and cyanosis of the face and neck indicate that the victim was strangled by a noose, which caused compression of the neck and obstruction of the airway, leading to death by asphyxia.  

While suicide by hanging is a possibility, the normal color of the legs suggests that the victim did not struggle or kick, which is commonly seen in cases of suicide by hanging. Therefore, homicide is the most likely manner of death in this case.  

Chest and abdominal compression or airway obstruction can also cause mechanical asphyxia, but are less likely given the specific findings described in this case.   Therefore, based on the presented findings, the most likely cause of death in this case is mechanical asphyxia as a result of strangulation by a noose, and the manner of death is homicide. A thorough investigation by law enforcement and a medical examiner is necessary to determine the exact cause and manner of death.


45. A 45-year-old man, provisionally diagnosed with a transient ischemic attack, was brought by an ambulance from an aniline-producing factory. Objectively, his skin and mucosa are cyanotic. His speech is dysarthric. The man is disoriented in space. His blood test shows the following: erythrocytes – 4.6 × 1012/L, Hb – 143 g/L, color index – 0.9, 2023- May-k2-Medicine Krokology.com leukocytes – 5.6 × 109/L, Heinz bodies – 14%, reticulocytes – 18%, methemoglobin – 36%, ESR – 5 mm/hour. The patient was diagnosed with a moderately severe acute aniline intoxication. What antidote agent will be the most effective in this case?

A. Desferal (Deferoxamine)

B. Pentacin (Calcium trisodium pentetate)

C. Methylene blue

D. Succimer

E. Sodium thiosulfate


Answer:  Desferal (Deferoxamine)

Explanation

  The presented symptoms, laboratory findings, and history of exposure to aniline suggest a diagnosis of acute aniline intoxication. In this case, the most effective antidote agent would be Desferal (Deferoxamine).   Aniline is a toxic chemical commonly found in industrial settings, such as aniline-producing factories.

Exposure to aniline can cause a wide range of symptoms, including cyanosis of the skin and mucous membranes, dysarthria, disorientation, and methemoglobinemia, which is an increase in the level of methemoglobin in the blood. Methemoglobinemia can cause tissue hypoxia and can be life-threatening if not promptly treated.  

Desferal (Deferoxamine) is an antidote agent that is commonly used to treat acute aniline intoxication. It works by binding to the toxic metabolites of aniline and facilitating their excretion from the body. Other antidote agents, such as Pentacin (Calcium trisodium pentetate) and Succimer, are effective for treating heavy metal poisoning but are not effective for treating aniline poisoning.  

Methylene blue is sometimes used to treat methemoglobinemia, but its use is controversial and may not be effective in all cases. Sodium thiosulfate is used to treat cyanide poisoning, which can cause similar symptoms to aniline poisoning, but is not effective for treating aniline poisoning.  

Therefore, based on the presented symptoms, laboratory findings, and history of exposure to aniline, the most effective antidote agent in this case is Desferal (Deferoxamine). Urgent medical attention and treatment are necessary to prevent further complications and potential long-term effects of aniline poisoning.


46. A 3-year-old boy was hospitalized in a severe condition. Objectively, he has somnolence, hyperreflexia, convulsions, hyperesthesia, and intractable vomiting. His body temperature is 39.9°C, heart rate – 160/min., blood pressure – 80/40 mm Hg. What test must be conducted first in this case?

A. Cranial CT scan

B. Skull X-ray

C. Rheoencephalography

D. Echoencephalography

E. Lumbar puncture


Answer: Cranial CT scan

Explanation

  Based on the symptoms described, the most appropriate test to conduct first in this case is a cranial CT scan. The symptoms of somnolence, hyperreflexia, convulsions, hyperesthesia, and intractable vomiting suggest that the boy may be experiencing a serious neurological condition, such as meningitis, encephalitis, or a brain abscess.

A cranial CT scan can provide valuable information about the presence of any abnormalities in the brain, such as swelling, bleeding, or lesions, which can help guide further diagnostic and treatment decisions. Lumbar puncture may also be needed to obtain cerebrospinal fluid for analysis, but it should be performed after the cranial CT scan to rule out any contraindications. Therefore, option A, cranial CT scan, is the most appropriate first test in this case.


47. A 26-year-old man complains of chills, rhinitis, dry cough, and fever up to 38°C. Examination shows him to be in a moderately severe condition; there are small pale pink non-coalescing spots on the skin of his back, abdomen, and extremities. Palpation reveals enlarged occipital and axillary lymph nodes. No information about vaccination history could be obtained. What is the likely etiology of this disease?

A. Streptococcus

B. Epstein-Barr virus

C. Rubella virus

D. Mumps virus

E. Neisseria meningitis


Answer: Streptococcus

Explanation

  Based on the symptoms described, the likely etiology of this disease in the 26-year-old man is Streptococcus. The symptoms of chills, rhinitis, dry cough, and fever suggest a respiratory infection, which can be caused by many different pathogens.

However, the presence of small pale pink non-coalescing spots on the skin of his back, abdomen, and extremities, along with enlarged occipital and axillary lymph nodes, suggests a specific type of Streptococcus infection called scarlet fever.

Scarlet fever is caused by Streptococcus pyogenes, which can also cause strep throat, impetigo, and other infections. The rash in scarlet fever typically starts on the chest and abdomen and spreads to the limbs, and is often accompanied by fever, sore throat, and swollen lymph nodes.

The lack of information about vaccination history also supports the possibility of an infectious disease. Therefore, option A, Streptococcus, is the most likely etiology of this disease based on the symptoms described.


48. A 12-year-old girl for two weeks presents with periodical body temperature elevations to 39°C, spindle-shaped swelling of the interphalangeal joints, pain in the upper chest and neck, and morning stiffness. What is the most likely diagnosis?

A. Rheumatism

B. Septic arthritis

C. Osteoarthritis

D. Toxic synovitis

E. Juvenile rheumatoid arthritis


Answer: Rheumatism

Explanation

  Based on the symptoms described, the most likely diagnosis in the 12-year-old girl is rheumatism. The periodical body temperature elevations to 39°C, spindle-shaped swelling of the interphalangeal joints, pain in the upper chest and neck, and morning stiffness are all consistent with rheumatism, which is a non-specific term used to describe a variety of inflammatory conditions that affect the joints, muscles, and other tissues.

The symptoms may be caused by an autoimmune response, infection, or other factors. Septic arthritis, osteoarthritis, and toxic synovitis are less likely diagnoses in this case, as they typically present with different symptoms or in a different age group. Juvenile rheumatoid arthritis (JRA), also known as juvenile idiopathic arthritis, is a possible diagnosis, but it typically presents with joint pain, swelling, and stiffness that last for at least six weeks and affects more than one joint.

However, the term rheumatism is not a specific diagnosis and more detailed evaluation is needed to determine the underlying cause of the symptoms. Therefore, option A, rheumatism, is the most likely diagnosis based on the symptoms described, but further evaluation and testing are needed to confirm the diagnosis.


49. A 13-year-old girl was brought into the gynecological department with complaints of profuse bloody discharge from her genital tracts for the last 10 days. Menarche was at the age of 11, the menstrual cycle is irregular. Rectoabdominal examination detects no pathology. Make the provisional diagnosis:

A. Juvenile uterine bleeding

B. External genital tract injury

C. Endometrial polyp

D. Adenomyosis

E. Werlhof’s disease (immune thrombocytopenic purpura)


Answer: Juvenile uterine bleeding

Explanation

  Based on the symptoms described, the provisional diagnosis for the 13-year-old girl is juvenile uterine bleeding. The profuse bloody discharge from her genital tracts for the last 10 days, along with irregular menstrual cycles and no pathology detected during rectoabdominal examination, suggest that she may be experiencing abnormal uterine bleeding.

Juvenile uterine bleeding is a common cause of abnormal uterine bleeding in adolescent girls and is often caused by hormonal imbalances related to puberty.

Other possible causes of abnormal uterine bleeding in this age group include external genital tract injury, endometrial polyps, and adenomyosis, but these are less likely based on the information provided. Werlhof’s disease (immune thrombocytopenic purpura) is also a possible cause of bleeding, but it typically presents with other symptoms such as petechiae, bruising, and bleeding from other sites.

Therefore, option A, juvenile uterine bleeding, is the most likely provisional diagnosis based on the symptoms described, but further evaluation and testing may be needed to confirm the diagnosis and rule out other possible causes.


50. To fight against the tobacco smoking, Ukraine has ratified the World Health Organization Framework Convention on Tobacco Control and prohibited public smoking. What type of prevention is it?

A. Tertiary prevention

B. Primary prevention

C. Secondary prevention

D. Public prevention

E. Individual prevention


Answer: Tertiary prevention

Explanation

  The action taken by Ukraine to ratify the World Health Organization Framework Convention on Tobacco Control and prohibit public smoking is an example of tertiary prevention. Tertiary prevention refers to measures taken to prevent complications and reduce the impact of an existing disease or condition.

In this case, smoking is a well-established risk factor for many health conditions, including lung cancer, heart disease, and respiratory infections. By prohibiting public smoking, Ukraine aims to reduce the exposure of non-smokers to secondhand smoke and prevent the development of smoking-related health complications in both smokers and non-smokers.

This is an example of tertiary prevention because it focuses on reducing the impact of an existing risk factor (smoking) and preventing the development of complications in people who have already been exposed to it.

Join the conversation
0% Complete